LSAT and Law School Admissions Forum

Get expert LSAT preparation and law school admissions advice from PowerScore Test Preparation.

User avatar
 Dave Killoran
PowerScore Staff
  • PowerScore Staff
  • Posts: 5849
  • Joined: Mar 25, 2011
|
#71214
This game is discussed in our Podcast: LSAT Podcast Episode 31: The September 2019 LSAT Logic Games Section


Complete Question Explanation
(The complete setup for this game can be found here: https://forum.powerscore.com/lsat/viewtopic.php?t=31306)

The correct answer choice is (E).

If you made a hypothetical in question #9 (or in the setup), this is another question that is easily answered. As we showed in #9, answer choice (E) is clearly possible:

  • J/P ..... G/N ..... _S_ ..... N/G ..... P/J ..... _M_ ..... _H_
     1 .....          2 .....    3 .....       4 .....      5 .....      6 .....      7

Answer choice (A): Accounting for the N :longline: HM sequence does not leave enough room to separate P and S.

Answer choice (B): As discussed during the setup, H cannot be in case 1, and so this answer cannot occur. Even if it could, per the first rule this leaves no viable artifact for case 7.

Answer choice (C): As discussed during the setup, H cannot be in case 1, and so this answer cannot occur.

Answer choice (D): If J is in case 1, then H must be in case 7, and according to the third rule H and M are next to each other. Thus, there is no way M could be in case 3.

Answer choice (E): This is the correct answer choice.

Get the most out of your LSAT Prep Plus subscription.

Analyze and track your performance with our Testing and Analytics Package.